What is Inequality: Definition and 1000 Discussions

In mathematics, the triangle inequality states that for any triangle, the sum of the lengths of any two sides must be greater than or equal to the length of the remaining side. This statement permits the inclusion of degenerate triangles, but some authors, especially those writing about elementary geometry, will exclude this possibility, thus leaving out the possibility of equality. If x, y, and z are the lengths of the sides of the triangle, with no side being greater than z, then the triangle inequality states that




z

x
+
y
,


{\displaystyle z\leq x+y,}
with equality only in the degenerate case of a triangle with zero area.
In Euclidean geometry and some other geometries, the triangle inequality is a theorem about distances, and it is written using vectors and vector lengths (norms):






x

+

y





x


+


y


,


{\displaystyle \|\mathbf {x} +\mathbf {y} \|\leq \|\mathbf {x} \|+\|\mathbf {y} \|,}
where the length z of the third side has been replaced by the vector sum x + y. When x and y are real numbers, they can be viewed as vectors in R1, and the triangle inequality expresses a relationship between absolute values.
In Euclidean geometry, for right triangles the triangle inequality is a consequence of the Pythagorean theorem, and for general triangles, a consequence of the law of cosines, although it may be proven without these theorems. The inequality can be viewed intuitively in either R2 or R3. The figure at the right shows three examples beginning with clear inequality (top) and approaching equality (bottom). In the Euclidean case, equality occurs only if the triangle has a 180° angle and two 0° angles, making the three vertices collinear, as shown in the bottom example. Thus, in Euclidean geometry, the shortest distance between two points is a straight line.
In spherical geometry, the shortest distance between two points is an arc of a great circle, but the triangle inequality holds provided the restriction is made that the distance between two points on a sphere is the length of a minor spherical line segment (that is, one with central angle in [0, π]) with those endpoints.The triangle inequality is a defining property of norms and measures of distance. This property must be established as a theorem for any function proposed for such purposes for each particular space: for example, spaces such as the real numbers, Euclidean spaces, the Lp spaces (p ≥ 1), and inner product spaces.

View More On Wikipedia.org
  1. M

    Solving an inequality for a change of variables

    Hi, This is as part of a larger probability change of variables question, but it was this part that was giving me problems. Question: If we have ## 0 < x_1 < \infty ## and ## 0 < x_2 < \infty ## and the transformations: ## y_1 = x_1 - x_2 ## and ## y_2 = x_1 + x_2 ##, find inequalities for...
  2. anemone

    MHB Can the Inequality of the Sum be Proven Using the Cube Root of -1?

    Assume that $x_1,\,x_2,\,\cdots,\,x_n\ge -1$ and $\displaystyle \sum_{i=1}^n x_i^3=0$. Prove that $\displaystyle \sum_{i=1}^n x_i\le \dfrac{n}{3}$.
  3. S

    MHB Can a Constant be Chosen to Satisfy an Inequality for All Real Numbers?

    Prove or disprove the following: There exists $A$ such that for all $a>0$ there exists $b>0$ such that for all $ x$: $|x-\ x_0|<b$ i mplies. $|\frac{1}{[x]}-A|<a$ where [x] is the floor value of x Gvf
  4. anemone

    MHB Inequality involving positive real numbers

    Prove that $\dfrac{y^2z}{x}+y^2+z\ge\dfrac{9y^2z}{x+y^2+z}$ for all positive real numbers $x,\,y$ and $z$.
  5. M

    I Do I understand Bell's Inequality Test correctly?

    Hi, I've been looking at Bells inequality test. To see if I understand it correctly I'd like to state it in my own words. Could you please let me know of I have it right? Thanks Michael We have 4 measurements, A,B,C,D Each measurement is True or False A is 0 degrees B is 45 degrees C is 25.5...
  6. S

    Solution of inequality of composite function involving inverse

    I can solve (i), I got x = -1.6 For (ii), I did like this: $$(f^{-1} o ~g)(x)<1$$ $$g(x)<f(1)$$ But it is wrong, the correct one should be ##g(x) > f(1)##. Why? Thanks
  7. jambaugh

    I An Alternative form of Bell's Inequality

    Bell's inequality in it's original form is: |cor(a,b) - cor(a,c)| \le 1 - cor(b,c) where ##a,b## and ##c## are random variables with values ##\pm 1##, and the correlation is then simply the expectation value of their products, ##cor(a,b)=E[ab]## or as usually expressed ##\langle ab\rangle##...
  8. C

    Finding a domain for a function

    I am having some trouble find the domain with this function: ##f(x)=\frac{1}{\sqrt{x^2-4x\cos(\theta)+4}}## and ##\theta\in[0,\pi]##.I know that the denominator needs to be greater than 0. So ##\sqrt{x^2-4x\cos(\theta)+4}>0##. I squared both side of the inequality, ##x^2-4x\cos(\theta)+4>0##...
  9. anemone

    MHB Solve Integer & Inequality: $x=(x-1)^3$ for $N$

    Let $x$ be a real number such that $x=(x-1)^3$. Show that there exists an integer $N$ such that $-2^{1000}<x^{2021}-N<2^{-1000}$.
  10. docnet

    Find the set of all functions that satisfy the inequality

    Problem: Find the set of all harmonic functions ##u(x,y,z)## that satisfy the following inequality in all of ##R^3## $$|u(x,y,z)|\leq A+A(x^2+y^2+z^2)$$ where ##A## is a nonzero constant. Work: I removed the absolute value bars by re-writing the expression $$-C-C(x^2+y^2+z^2)\leq u\leq...
  11. K

    How to represent this absolute value inequality with constants?

    see attached image, it asks to repesent it in x-graph constant "a" isn't conditioned. Do I need to separate it into a few cases of the constant a and represent each in one x-graph?
  12. O

    I Bell's Inequality => 4 entangled Photons impossible?

    Correlation between polarization measurements of entangled photons at angles less than 45 are greater than classically statistically possible. No set of hidden variables can be preordained to explain the 75% correlation of photon measurements at 30 degrees and complete anticorrelation of...
  13. M

    I To understand Bell's inequality

    First I give detail of what I think I understood so far. Suppose, there are three angles A, B, C separated by 120° angles. A can measure + (spin up in A direction, we call it A+), and - (spin down in A direction, we call it A-). Same goes for B+, B- and C+, C-. I have choose A direction to...
  14. D

    Inequality proof: If a>b implies a>c then b>c

    Summary:: To prove a conditional statement on a pair of inequalitites. Mentor note: Moved from technical forum section, so the post is missing the usual fields. I feel it should be possible to prove this but I keep getting lost in the symbolic manipulation. Theorem: If a>b implies a>c then...
  15. anemone

    MHB How Can We Prove the Summation Inequality for a Given Sequence?

    Assume that $x_1,\,x_2,\,\cdots,\,x_n \ge -1$ and $\displaystyle \sum_{i=1}^n x_i^3=0$. Prove that $\displaystyle \sum_{i=1}^n x_i \le \dfrac{n}{3}$.
  16. anemone

    MHB Real Number Pairs $(p,\,q)$ Satisfying Inequality

    Find all pairs of real numbers $(p,\,q)$ such that the inequality $|\sqrt{1-x^2}-px-q|\le \dfrac{\sqrt{2}-1}{2}$ holds for every $x\in [0,\,1]$.
  17. S

    MHB Proof of Triangle Inequality for $n$ Natural Numbers

    Prove for all $n\in N$ $\dfrac{|a_1+...a_n|}{1+|a_1+...+a_n|}\leq\dfrac{|a_1|}{1+|a_1|}+...\dfrac{|a_n|}{1+|a_n|}$
  18. anemone

    MHB Can the Sum of Two Trigonometric Functions Be Less than pi/2?

    Prove that $\cos(\sin x))+\cos(\cos x))<\dfrac{\pi}{2}$.
  19. anemone

    MHB Inequality of positive real numbers

    If $x$ and $y$ are positive real numbers, prove that $4x^4+4y^3+5x^2+y+1\ge 12xy$.
  20. anemone

    MHB Inequality with positive real numbers a and b

    Let $a$ and $b$ be positive real numbers such that $a+b=1$. Prove that $a^ab^b+a^bb^a\le 1$.
  21. J

    Confused with this proof for the Cauchy Schwarz inequality

    Im confused as finding the minimum value of lambda is an important part of the proof but it isn't clear to me that the critical point is a minimum
  22. greg_rack

    Doubt solving a polynomial inequality

    I got this function in a function analysis and got confused on how to solve its positivity; I rewrote it as: $$\sqrt{x^{2}-2x}>x-1 \rightarrow x^2-2x>x^2-2x+1$$ And therefore concluded it must've been impossible... but I'm certainly missing something stupid, since plotting the graphs of the two...
  23. mcastillo356

    B The same inequality, but two equivalent ways to express it

    I've been given this answer: to move from negative to positive angles, ##-\theta'=2\pi-\theta##; and to move from positive to negative angles, ##\theta'=\theta-2\pi##. But my question is if there is any way to calculate it in a sequence of inequalities' steps. If I am being cumbersome, forgive...
  24. S

    MHB Basic Inequality Prove: $A\leq B\wedge B\leq A \Rightarrow A=B$

    Prove: $A\leq B\wedge B\leq A\Rightarrow A=B$
  25. yucheng

    Linear Equations: how to deduce this inequality is true?

    The solution from my book: From $$\frac{3}{x+2}<\frac{13}{12}<\frac{3}{x} \tag1$$ It follows that ##13x<36<13(x+2)## x<3, i.e. x = 1 or 2. By checking, x=1 is not the solution and x = 2 satisfies the equation. However, how does the author deduce (1)?
  26. brotherbobby

    Solving a modulus inequality in ##x##

    The answer to the above problem is baffling, despite its straightforward nature. I will post the answer later, but here is my solution first. Solution : (1) ##2x+1 > x## : In this case, we have ##2x - x > -1 \Rightarrow \boxed{x > - 1}## (2) ##2x+1 < -x## : In this case, we have ##2x+x < -1...
  27. anemone

    MHB Proving $\dfrac{3}{2}$ Inequality in Acute Triangle $ABC$

    In an acute triangle $ABC$, prove that $\dfrac{\cos A}{\cos (B-C)}+\dfrac{\cos B}{\cos (C-A)}+\dfrac{\cos C}{\cos (A-B)}\ge \dfrac{3}{2}$.
  28. S

    MHB Proof of Inequality: $|a+b| \leq |a| + |b|$

    prove the following inequality: $\dfrac{|a+b|}{1+|a+b|}$ $\leq \dfrac{|a|}{1+|a|}$ +$\dfrac{|b|}{1+|b|}$
  29. anemone

    MHB Prove Inequality: $\sqrt{ab}+\sqrt{cd}\le \sqrt{(a+d)(b+c)}$

    Prove that for positive reals $a,\,b,\,c,\,d$, $\sqrt{ab}+\sqrt{cd}\le \sqrt{(a+d)(b+c)}$.
  30. C

    Simple Induction Help with Lemma for proof of AM-GM inequality

    Summary:: x Hey, I'm learning calculus and had to prove the following Lemma which is used to prove AM-GM inequality, I had tried to prove it on my own and it is quite different from what is written in my lecture notes. I have a feeling that my proof of the Lemma is incorrect, but I just don't...
  31. anemone

    MHB Can you prove this inequality challenge?

    Prove that $\sqrt[n]{1+\dfrac{\sqrt[n]{n}}{n}}+\sqrt[n]{1-\dfrac{\sqrt[n]{n}}{n}}<2$ for any positive integer $n>1$.
  32. anemone

    MHB Proving Series Inequality: $\sqrt[3]{\frac{2}{1}}$ to $\frac{1}{8961}$

    Prove that $\sqrt[3]{\dfrac{2}{1}}+\sqrt[3]{\dfrac{3}{2}}+\cdots+\sqrt[3]{\dfrac{996}{995}}-\dfrac{1989}{2}<\dfrac{1}{3}+\dfrac{1}{6}+\cdots+\dfrac{1}{8961}$.
  33. anemone

    MHB Can You Solve This Challenging Inequality Problem?

    Given positive real numbers $a,\,b,\,c$ and $d$ that satisfy the following inequalities: $a \le 1 \\a+4b \le 17\\a+4b+16c \le273\\a+4b+16c+64d \le4369$ Find the minimum value of $\dfrac{1}{d}+\dfrac{2}{4c+d}+\dfrac{3}{16b+4c+d}+\dfrac{4}{64a+16b+4c+d}$.
  34. MidgetDwarf

    Help with this inequality needed for a proof in a textbook

    Note that if we prove problem 4, the proof for problem 5 follows directly. We use properties of logarithms to combine the right hand side of ln into a single logarithm. Then we raise both side of the inequality to a power of e. Which leads us to the desired inequality. But, when I try to be...
  35. anemone

    MHB Prove Triangle Inequality: $\frac{a}{\sqrt[3]{4b^3+4c^3}}+...<2$

    Let $a,\,b$ and $c$ be the side lengths of a triangle. Prove that $\dfrac{a}{\sqrt[3]{4b^3+4c^3}}+\dfrac{c}{\sqrt[3]{4a^3+4b^3}}+\dfrac{a}{\sqrt[3]{4b^3+4c^3}}<2$.
  36. anemone

    MHB What is the Trigonometric Inequality for $0<x<\dfrac{\pi}{2}$?

    Show that for all $0<x<\dfrac{\pi}{2}$, the following inequality holds: $\left(1+\dfrac{1}{\sin x}\right)\left(1+\dfrac{1}{\cos x}\right)\ge 5\left[1+x^4\left(\dfrac{\pi}{2}-x\right)^4\right]$
  37. GodfreyHW

    I Inequality from a continuity exercise

    I am reading from Courant's book. He gave an example of the continuity of ##f(x)=5x+3## by finding ##\delta=\epsilon/5##. He then said that ##|x-x_0|## does not exceed ##|y-y_0|/5##, but I don't see how he came up with this inequality. I know that ##|x-x_0|<\epsilon/5##, and that...
  38. anemone

    MHB Inequality involving area under a curve

    Prove that for every $x\in (0,\,1)$ the following inequality holds: $\displaystyle \int_0^1 \sqrt{1+(\cos y)^2} dy>\sqrt{x^2+(\sin x)^2}$
  39. anemone

    MHB Inequality involving a, b, c and d

    Given the real numbers $a,\,b,\,c$ and $d$, prove that $(1+ab)^2+(1+cd)^2+a^2c^2+b^2d^2\ge 1$
  40. MrMoe

    Bearing race diameter inequality question

    Hello all this is my first post. I am not an engineer, but I wish I was. I have been enjoying watching Dan Gelbart!s YouTube Chanel. His air bearing has me wondering about the relationship and apparent conflict inside ball bearings. How does the ball travel around the shorter distance...
  41. M

    Find the set of points that satisfy:|z|^2 + |z - 2*i|^2 =< 10

    Hello everyone, I've been struggling quite a bit with this problem, since I'm not sure how to approach it correctly. The inequality form reminds me of the equation of a circle (x^2 + y^2 = r^2), but I have no idea how to be sure about it. Would it help just to simplify the inequality in terms...
  42. Opalg

    MHB Can You Prove This Inequality Challenge?

    In a recent https://mathhelpboards.com/threads/inequality-challenge.27634/#post-121156, anemone asked for a proof that $1-x + x^4 - x^9 + x^{16} - x^{25} + x^{36} > 0$. When I graphed that function, I noticed that in fact it is never less than $\frac12$. If you add more terms to the series, this...
  43. anemone

    MHB Inequality Challenge: Prove $x$ for $x>0$

    Prove $x+x^9+x^{25}<1+x^4+x^{16}+x^{36}$ for $x>0$.
  44. anemone

    MHB A quite delicious inequality problem

    Prove that $2^{2\sqrt{3}}>10$.
  45. wael_khayati

    How to Complete a Linear Inequality Assignment Without Full Instructions?

    hey so, this is an algebra assignment that we had to do and i really didn't understand the course material that well, but i managed to do the very first steps. anyways i was hoping you guys could help me finish the rest of this table. https://ufile.io/jowwrfj3 or you can see the file attached
  46. anemone

    MHB Inequality of cubic and exponential functions

    Prove that $3^n\ge(n+3)^3$ for any natural number $n\ge6$.
  47. jk22

    I CHSH inequality : renaming and absolute values

    In some derivations of the CHSH inequality, https://en.m.wikipedia.org/wiki/CHSH_inequality, the following arises : $$CHS=\int A(a,l1)B(b,l1)dl1-\int A(a,l2)B(b',l2)dl2+\int A(a',l3)B(b,l3)dl3+\int A(a',l4)B(b',l4)dl4\\ =\int A(a,l)B(b,l)dl1-A(a,l)B(b',l)+A(a',l)B(b,l)+A(a',l)B(b',l)dl$$ 1)...
  48. C

    MHB Inequality involving Gaussian integral

    I'm trying to solve the inequality: $$ \int \limits_0^1 e^{-x^2} \leq \int \limits_1^2 e^{x^2} dx $$I know that $\int \limits_0^1 e^{-x^2} \leq 1$, but don't see how to take it from there. Any ideas?
  49. K

    B Quick question about this inequality

    For concreteness, let me consider real numbers. If ##A > B## and ##B \sim C##, does it mean that ##A \sim C##? If instead ##B = C##, then obviously that wouldn't imply ##A = C##.
  50. karush

    MHB *gre.al.9 GRE Exam Inequality with modulus or absolute value

    given $|y+3|\le 4$ we don't know if y is plus or negative so $y+3\le 4 \Rightarrow y\le 1$ and $-(y+3)\le 4$ reverse the inequality $ y+3 \ge -4$ then isolate y $y \ge -7$ the interval is $-7 \le y \le 1$
Back
Top